Search results
Create the page "AoPS:T" on this wiki! See also the search results found.
- ...px; padding-bottom:10px;"> <span style="font-size: 200%">'''Welcome to the AoPS Wiki!'''</span> The AoPS Wiki project is administered by the [[Art_of_Problem_Solving|<span style="c2 KB (257 words) - 15:58, 13 February 2025
- Many AoPS Community members and online school students have been participants at Nati ...ions serve as a selection filter for state competitions. A few states don't need to host chapter competitions due to a small population size.10 KB (1,503 words) - 22:32, 4 February 2025
- [[AoPS]] hosts [http://www.artofproblemsolving.com/Forum/index.php?f=298 middle sc ...groups.io/g/CMP-mathclub/topic/mathalon/93230445 MD Mathalon] October (don't need to go through school)2 KB (264 words) - 10:23, 29 September 2024
- ...ded to high scorers at the end of the year. These typically include a free t-shirt, along with other prizes like books or software of the participant's * [http://artofproblemsolving.com/community/c123_usamts USAMTS Forum] on [[AoPS]]4 KB (613 words) - 21:12, 15 October 2024
- ...[[Mathematical Association of America]] (MAA). [[Art of Problem Solving]] (AoPS) is a proud sponsor of the MAA. ...[[IMO]], but had little support. After she published the article "Why can't we have a USA Mathematical Olympiad?" in the ''American Mathematical Monthl6 KB (836 words) - 03:57, 3 February 2025
- https://youtu.be/6xNkyDgIhEE?t=1699 ...result <math>T</math>. Find <math>R = (T - K)/2</math>. Check: Is <math>T = R \cdot 80</math>.10 KB (1,572 words) - 21:11, 22 September 2024
- These two examples use options specifically based on digits, but this isn't the entire picture of constructive counting. The following example uses con ...ce the three <math>B</math>s can go is in the three empty boxes, so we don't have to account for them after choosing the <math>A</math>s. Thus, there ar13 KB (2,018 words) - 14:31, 10 January 2025
- ...entary counting''' is a [[counting]] method where one counts what they don't want, then subtracts that from the total number of possibilities. In proble ...e integers, with no restrictions, is <math>99</math> integers. What we don't want are the multiples of five. These are <math>5, 10,..., 95</math> or <ma8 KB (1,192 words) - 16:20, 16 June 2023
- ...n fact, we wouldn't want any fractions). The [[Roman system]], which didn't have any base system at all, used certain letters to represent certain valu * The AoPS [http://www.artofproblemsolving.com/Books/AoPS_B_Item.php?page_id=10 Introd4 KB (547 words) - 16:23, 30 December 2020
- ...//www.artofproblemsolving.com/Forum/index.php?f=346\ Matrix-Linear Algebra AOPS forum] *[http://www.artofproblemsolving.com/Forum/viewtopic.php?t=89911\ This is a thread about what vectors are.]11 KB (1,875 words) - 14:35, 7 January 2025
- <!-- taken from AoPS Vol. 2 --> ...iv 1\mod 5</math> implies <math>z_0</math> is even. QED [Oops, this doesn't work. 21 (or <math>3^4 = 81</math>) are equal to <math>1\mod 5</math> and9 KB (1,434 words) - 00:15, 4 July 2024
- ...the [https://artofproblemsolving.com/community/c594864_aops_mock_contests AoPS Mock Contests forum]. * Pair up with another user on AoPS and write it together. Two minds are much better than one. With just one51 KB (6,175 words) - 20:41, 27 November 2024
- https://youtu.be/7an5wU9Q5hk?t=777 We don't always need to perform tedious computations to discover solutions to intere16 KB (2,410 words) - 13:05, 3 January 2025
- ...bset as soon as they arrived and not typed any others. Since <math>\mathrm{T}</math> has 8 elements, it has <math>2^{8}=256</math> subsets (including th <math>\textbf{Case 2:}</math> Since letter 9 didn't arrive before lunch, the question is: Where can it be inserted in the typin7 KB (1,188 words) - 07:02, 15 August 2024
- ~MRENTHUSIASM (credit given to AoPS) ...ath> to this form by exploiting the other properties. The second one doesn't help us immediately, so we will use the third one.4 KB (538 words) - 12:24, 12 October 2021
- ...lus one extra vertex since once again, the vertex you started on, it doesn't loop around, so it must end at another vertex). Hence the answer is <math>n ...Sorry if it didn't make sense. Maybe if you find some way to contact me on aops, I could try and help). (For instance, I could see possible confusion at th9 KB (1,659 words) - 17:35, 20 June 2024
- * <url>viewtopic.php?t=84548 Discussion on AoPS/MathLinks</url>3 KB (506 words) - 16:54, 22 June 2023
- * <url>viewtopic.php?t=84550 Discussion on AoPS/MathLinks</url>2 KB (398 words) - 08:48, 5 August 2014
- A theorem of T. Nagell implies that if <math>d(g_i) \ge 2</math> the ratio * <url>viewtopic.php?t=84553 Discussion on AoPS/MathLinks</url>9 KB (1,699 words) - 12:48, 11 April 2020
- ...ath> or <math>(2^{i_0-m} - 1)/2 < t < (2^{i_0-m} + 1)/2</math>. Thus <math>t = 2^{i_0-m-1}</math> and the jump over <math>2^{i_0}</math> is from <math>2 ...the distinct <math>y_j</math> form a valid path for the frog. If <math>j = t</math> the congruence gives <math>y_t\equiv x_t\equiv 0\pmod{2^{r+1}}</math7 KB (1,280 words) - 16:23, 26 March 2016